Đến nội dung

Hình ảnh

USAMO 2017 ngày 1


  • Please log in to reply
Chủ đề này có 9 trả lời

#1
lamNMP01

lamNMP01

    Hạ sĩ

  • Thành viên
  • 96 Bài viết

Bài 1: CMR tồn tại vô hạn cặp $(a,b)$ thoả mãn $a$, $b$ nguyên tố cùng nhau và đồng thời lớn hơn 1 để $a^b+b^a$ chia hết cho $a+b$

 

 

Bài 2: Cho $m_1, m_2, \ldots, m_n$ là n số nguyên dương không nhất thiết phân biệt . Với dãy các số nguyên bất kì $A = (a_1, \ldots, a_n)$ và 1 hoán vị bất k ì$w = w_1, \ldots, w_n$ của $m_1, m_2, \ldots, m_n$ định nghĩa $A$-inversion của $w$ là các  cặp $w_i$ và $w_j$ với $i$ < $j$ thoả mãn các điều kiện :

   i)  $a_i \ge w_i > w_j$

   ii) $w_j > a_i \ge w_i$ hoặc 

  iii)  $w_i > w_j > a_i$

 CMR với 2 dãy số nguyên $A = (a_1, \ldots, a_n)$  và $B = (b_1, \ldots, b_n)$ và với mỗi số nguyên dương $k$ ,số hoán vị của $m_1, \ldots, m_n$ có đúng k $A$-inversion bằng với số bộ hoán vị $m_1, \ldots, m_n$ có đúng $k$ $B$-inversion.

 

Bài 3: Cho $ABC$ là tam giác không có 3 cạnh bằng nhau , nội tiếp $\Omega$ và có tâm nội tiếp $I$. Tia $AI$ cắt $BC$ tại $D$ và $\Omega$ lần thứ 2 tại M. Đường tròn đường kính $DM$ cắt $\Omega$ tại $K$ . $MK$ cắt $BC$ tại $S$. Lấy $N$ là trung điểm của tia $IS$. Đường tròn ngoại tiếp $\triangle KID$ và $\triangle MAN$ cắt nhau lần lượt tại $L_1$ và $L_2$. CMR $\Omega$ đi qua trung điểm của $IL_1$ hoặc $IL_2$.

 

 

                                                                           Chúc mọi người có 1 buổi đêm vui vẻ :)



#2
dogsteven

dogsteven

    Đại úy

  • Thành viên
  • 1567 Bài viết

Bài 3. Gọi $J$ là tâm đường tròn bàng tiếp góc $A$ của tam giác $ABC$ thì $M$ là trung điểm $IJ$

Dễ thấy rằng $AS$ là phân giác ngoài góc $A$ của tam giác $ABC$ nên $AS$ đi qua $G$ là trung điểm cung $BC$ của $(ABC)$

Gọi $F$ là giao điểm của $(ASI)$ và $(IBC)$ thì $MN\perp IF$ tại trung điểm $H$ của $IF$.

Ta có $GA.GS = GB^2$ nên $G\in IF$, suy ra $H\in (ABC)$

Ta có $\widehat{FAN}=90^o-\widehat{AIG}=\widehat{AMN}$ nên $F\in (AMN)$

Ta còn có $\widehat{MDK}=\widehat{GSK}=\widehat{IFK}$ do $G,S,K,F$ đồng viên. Do đó $F\in (KID)$

Ta có điều phải chứng minh.


Bài viết đã được chỉnh sửa nội dung bởi dogsteven: 21-04-2017 - 10:16

Quyết tâm off dài dài cày hình, số, tổ, rời rạc.


#3
Donald Trump

Donald Trump

    Binh nhất

  • Thành viên mới
  • 28 Bài viết

Bài 1. Chọn $a$ lẻ, $a\equiv 2\pmod 3$ thì tồn tại vô hạn $a$ như vậy.

Xét bộ $(a,a^2+1)$ thì $a^2+1-a\equiv 1+1-2\equiv 0\pmod 3$ $\Rightarrow $ $a^2+a+1$ $\mid$ $a^3-1$ $\mid$ $a^{a^2+1-a}-1$ $\mid$ $a^{a^2+1}-a^a$

Do $a$ lẻ nên $a^2+a+1$ $\mid$ $(a^2+1)^a$ $+$ $a^{a}$ $\Rightarrow$ $a^2+a+1$ $\mid$ $a^{a^2+1}+(a^2+1)^a$

Mặt khác $(a,a^2+1)$ $=$ $1$ nên $(a,a^2+1)$ thỏa bài toán. $\square$

 



#4
JUV

JUV

    Trung sĩ

  • Điều hành viên OLYMPIC
  • 138 Bài viết

Bài 2:

Ta sẽ chứng minh rằng số hoán vị chứa đúng $k$ $A-inversion$ không phụ thuộc vào dãy $A$ (Và bằng số số hoán vị có đúng $k$ nghịch thế).

Bài toán hiển nhiên đúng với $n=1$, giả sử bài toán đúng với $n=k$, xét $n=k+1$. Xét $1$ hoán vị có chứa số $w_1$ đứng đầu dãy. Gọi $f$ là số số $w_i$ thoả mãn $(w_1;w_j)$ là $1$ $A-inversion$. Nếu $a_1>w_1$, số số $w_j$ thoả mãn điều kiện bằng số số $t$ trong $n$ số $m_1,m_2,...,m_n$ thoả mãn $t>a_1$ hoặc $t<w_1$, số các số đó hoàn toàn không phụ thuộc vào cách sắp xếp $n-1$ số cồn lại khác $w_1$. Tương tự với trường hợp $a_1<w_1;a_1=w_1$. Gọi số số $w_j$ lạp thành cặp $A-inversion$ với $w_1$ là $f$, ta đếm cách sắp xếp $n-1$ số còn lại để có đúng $k-f$ cặp, và nó không phụ thuộc vào $A-a_1$. Tương tự với các trường hợp những số khác xếp ở vị trí đầu trong hoán vị, tổng lại các cách sắp xếp thì ta được số cách sắp xếp để có đúng $k$ cặp $A-inversion$ và nó không phụ thuộc vào $A$. Mệnh đề đúng với $n=k+1$, $\Rightarrow$ $Q.E.D$


Bài viết đã được chỉnh sửa nội dung bởi JUV: 21-04-2017 - 21:02


#5
IHateMath

IHateMath

    Thượng sĩ

  • Thành viên
  • 299 Bài viết

Ngày $2$.

 

$4$. Cho $P_1,P_2,\dots ,P_{2n}$ là $2n$ điểm phân biệt trên đường tròn đơn vị $x^2+y^2=1$ và khác $(1,0)$. Mỗi điểm được tô đỏ hoặc xanh với đúng $n$ điểm đỏ và $n$ điểm xanh. Gọi $R_1,R_2,\dots, R_n$ là bất kỳ cách đánh dấu các điểm đỏ nào. Gọi $B_1$ là điểm xanh gần nhất với $R_1$ khi đi quanh đường tròn theo chiều ngược kim đồng hồ, xuất phát từ $R_1$. Sau đó gọi $B_2$ là điểm xanh gần nhất trong các điểm còn lại với $R_2$ khi đi quanh đường tròn theo chiều ngược kim đồng hồ, xuất phát từ $R_2$. Cứ như thế cho đến khi tất cả các điểm đều được đánh dấu $B_1, B_2,\dots ,B_{n}$. Chứng minh rằng số cung ngược chiều kim đồng hồ có dạng $R_iB_i$ mà chứa điểm $(1,0)$ thì không phụ thuộc với cách chọn thứ tự các điểm đỏ $R_1,R_2,\dots R_n$.

 

$5$. Gọi $\mathbf{Z}$ là tập tất cả các số nguyên dương. Tìm  tất cả các số thực dương $c$ sao cho tồn tại một cách đánh dấu các điểm nguyên $(x,y)\in\mathbf{Z}^2$ bằng các số nguyên dương mà:

$\bullet$ Chỉ có hữu hạn số nguyên dương được dùng để đánh dấu

$\bullet$ Với mỗi số nguyên dương $i$ được dùng để đánh dấu, khoảng cách giữa hai điểm cùng được đánh dấu $i$ ít nhất là $c^i$.

 

$6$. Tìm giá trị nhỏ nhất của

$$\frac{a}{b^3+4}+\frac{b}{c^3+4}+\frac{c}{d^3+4}+\frac{d}{a^3+4}$$

trong đó $a,b,c,d$ là các số thực không âm và $a+b+c+d=4$.

 

$$\text{HẾT}$$

 

Aww, bất đẳng thức again!



#6
canhhoang30011999

canhhoang30011999

    Thiếu úy

  • Thành viên
  • 634 Bài viết

6

ta có $\sum \frac{a} {b^{3}+4}= \sum \frac{a} {4} -\frac{ab^{3}} {4(b^{3}+4)} \geq \sum \frac{a} {4} - \frac{ab} {12}$

$=1-\frac{ab+bc+cd+da} {12} \geq 1- \frac{(a+c)(b+d)} {12} \geq \frac{2} {3}$

dấu bằng xảy ra khi $a=d=2, b=c=0$


Bài viết đã được chỉnh sửa nội dung bởi canhhoang30011999: 24-04-2017 - 08:33


#7
Mr Cooper

Mr Cooper

    Sĩ quan

  • Thành viên
  • 496 Bài viết

$\boxed{6}$

Dễ thấy rằng: $\dfrac{1}{x^3+4}=\dfrac{3-x}{12}+\dfrac{x(x-2)^2(x+1)}{12(x^3+4)}$
$\Rightarrow \sum \dfrac{a}{b^3+4}\ge \dfrac{3(a+b+c+d)-(a+c)(b+d)}{12}\ge \dfrac{2}{3}$
Sử dụng bất đẳng thức $\text{AM-GM}$ ta có:$(a+c)(b+d)\le \dfrac{(a+b+c+d)^2}{4}=4$
Hoàn tất chứng minh


Bài viết đã được chỉnh sửa nội dung bởi Mr Cooper: 24-04-2017 - 18:44


#8
dogsteven

dogsteven

    Đại úy

  • Thành viên
  • 1567 Bài viết

Bài 4.

1) Đầu tiên ta thấy rằng: Xét $(a)=(a_1, a_2, ..., a_n), (b)=(b_1, b_2, ..., b_n)$ là hai hoán vị bất kỳ của $(1, 2, 3, ..., n)$ thì bằng liên tiếp các bước đổi chỗ hai số có vị trí $(i, i+1)$ trong $(a)$ thì đến một lúc nào đó ta sẽ được $(b)$

Ta sẽ chọn số $a_k=b_1$ và đưa nó dần lên về vị trí đầu tiên, tương tự với các $b_2, b_3, ..., b_n$.

 

2) Ta sẽ chứng minh rằng: Khi đổi chỗ hai điểm $R_i, R_{i+1}$ thì kết quả bài toán không đổi.

Xét $B_i$ nằm trên cung $R_{i+1}R_{i}$ tính theo ngược chiều kim đồng hồ. Khi đó $B_{i+1}$ nằm trên cung $B_iR_{i}$ theo ngược chiều kim đồng hồ.

Khi đó ta đổi chỗ $R_i$ và $R_{i+1}$ thì $B_i$ và $B_{i+1}$ không đổi. Khi đó $(1, 0)$ dù ở vị trí nào cũng ra kết quả ban đầu.

Tương tự ta xét các trườn hợp còn lại.

 

Theo 1), với mỗi cách viết tùy ý $R_{i_1}, R_{i_2}, ...., R_{i_n}$ có thể được tạo ra nhờ các bước hoán vị liên tiếp các điểm có vị trí $j$ và $j+1$, bắt đầu từ $R_1, R_2, ..., R_n$

Theo 2), qua mỗi bước hoán vị hai điểm có vị trí liên tiếp thì kết quả không đổi.

 

Do đó ta có điều phải chứng minh.


Quyết tâm off dài dài cày hình, số, tổ, rời rạc.


#9
Uchiha sisui

Uchiha sisui

    Trung sĩ

  • Thành viên
  • 196 Bài viết

Bài 3.

 

Đoạn đầu có thể xử lý được bằng trực đẳng phương như sau:

 

Gọi $W,G$ lần lượt là trung điểm của $BC$, cung $BC$ chứa $A$.

 

Xét ba đường tròn $(DM)$, $(\varpi )$, $(ADWG)$ thì ta có trục đẳng phương của ba đường tròn đồng quy tại $S$

 

Do đó dễ chứng minh $SA$ là phân giác ngoài của tam giác ABC tại đỉnh A.

 

Còn đoạn sau mình xử lý giống bạn dogsteven.



#10
toanhoc2017

toanhoc2017

    Thiếu úy

  • Banned
  • 628 Bài viết

CÂU BĐT LÀ ĐỘ CHẾ THÔI






1 người đang xem chủ đề

0 thành viên, 1 khách, 0 thành viên ẩn danh